Difference between revisions of "2008 AMC 12B Problems/Problem 13"

(need solution)
(added box)
Line 10: Line 10:
 
\textbf{(D)}\ \frac{3-\sqrt3}{9} \qquad
 
\textbf{(D)}\ \frac{3-\sqrt3}{9} \qquad
 
\textbf{(E)}\ \frac{\sqrt3}{12}</math>
 
\textbf{(E)}\ \frac{\sqrt3}{12}</math>
 +
 +
==See Also==
 +
 +
{{AMC12 box|year=2008|ab=B|num-b=12|num-a=14}}

Revision as of 13:34, 30 May 2011

This problem needs a solution. If you have a solution for it, please help us out by adding it.

Problem

Vertex $E$ of equilateral $\triangle{ABC}$ is in the interior of unit square $ABCD$. Let $R$ be the region consisting of all points inside $ABCD$ and outside $\triangle{ABC}$ whose distance from $AD$ is between $\frac{1}{3}$ and $\frac{2}{3}$. What is the area of $R$?

$\textbf{(A)}\ \frac{12-5\sqrt3}{72} \qquad \textbf{(B)}\ \frac{12-5\sqrt3}{36} \qquad \textbf{(C)}\ \frac{\sqrt3}{18} \qquad \textbf{(D)}\ \frac{3-\sqrt3}{9} \qquad \textbf{(E)}\ \frac{\sqrt3}{12}$

See Also

2008 AMC 12B (ProblemsAnswer KeyResources)
Preceded by
Problem 12
Followed by
Problem 14
1 2 3 4 5 6 7 8 9 10 11 12 13 14 15 16 17 18 19 20 21 22 23 24 25
All AMC 12 Problems and Solutions